Administración     

Olimpiadas de Matemáticas
Página de preparación y problemas

Selector
La base de datos contiene 1154 problemas y 775 soluciones.
OME Local
OME Nacional
OIM
OME Andalucía
Retos UJA
+20
Problema 157
Dado un número natural $n$, determinar qué número es mayor: $n!$ ó $\sqrt{n^n}$.
pistasolución 1info
Pista. El problema es equivalente a comparar los números $(n!)^2$ y $n^n$. Ahora bien, el número $(n!)^2$ se puede expresar como $(1\cdot n)\cdot(2(n-1))\cdots((n-1)2)\cdot(n\cdot 1)$, ¿qué le ocurre a cada uno de los factores de la forma $k(n-k)$?
Solución. Si probamos con valores pequeños de $n$, para $n=1$ ó $n=2$ ambos son iguales pero para $n\gt 2$ parece ser que $n!$ siempre es mayor que $\sqrt{n^n}$, por lo que es razonable intentar probar que $n!\gt\sqrt{n^n}$ para todo $n\gt 2$. Elevando al cuadrado, esto equivale a probar que $(n!)^2\gt{n^n}$ y eso será lo que haremos.

Observemos que $(n!)^2=1^2\cdot2^2\cdots n^2$ y este producto lo podemos reordenar como el producto de $n$ factores (agrupando el primero con el último, el segundo con el penúltimo, etc.), es decir, \[(n!)^2=(1\cdot n)\cdot(2\cdot(n-1))\cdots((n-1)\cdot 2)\cdot(n\cdot 1).\] Ahora vamos a analizar cada término $k(n-k)$. Si nos fijamos bien, la función $f(x)=x(n-x)$ se anula en $x=0$ y $x=n$ y es positiva en el intervalo $(0,n)$. Como su gráfica es una parábola, es fácil ver que los valores más pequeños de $f(x)$ cuando $x$ es un número natural en el intervalo $[1,n-1]$ se alcanzarán para $x=1$ y $x=n-1$, lo que nos dice que $k(n-k)\geq f(1)=n$ para $k$ entre $1$ y $n-1$ y la igualdad se da sólo para $k=1$ y $k=n-1$.

En consecuencia, sustituyendo cada término de la forma $k(n-k)$ por $n$, deducimos que $(n!)^2\geq{n^n}$ y la igualdad se alcanza sólo cuando $n=1$ ó $n=2$.

Si crees que el enunciado contiene un error o imprecisión o bien crees que la información sobre la procedencia del problema es incorrecta, puedes notificarlo usando los siguientes botones:
Informar de error en enunciado Informar de procedencia del problema
Problema 150
Hallar todas las sucesiones $0\lt a_1\leq a_2\leq\ldots\leq a_n$ de numeros reales verificando \[\left\{\begin{array}{rcl} a_1+a_2+\ldots+a_n&=&96\\a_1^2+a_2^2+\ldots+a_n^2&=&144\\a_1^3+a_2^3+\ldots+a_n^3&=&216 \end{array}\right.\]
pistasolución 1info
Pista. Intenta utilizar la desigualdad de Cauchy-Schwarz involucrando a las cantidades del enunciado.
Solución. La desigualdad de Cauchy-Schwarz aplicada a los vectores \begin{eqnarray*} u&=&(a_1^{3/2},a_2^{3/2},\ldots,a_n^{3/2}),\\ v&=&(a_1^{1/2},a_2^{1/2},\ldots,a_n^{1/2}), \end{eqnarray*} nos dice que \[(a_1^2+a_2^2+\ldots+a_n^2)^2\leq(a_1^3+a_2^3+\ldots+a_n^3)(a_1+a_2\ldots+a_n).\] Observemos que en nuestro caso se da la igualdad puesto que $144^2=96\cdot216$ luego $u$ y $v$ son proporcionales, es decir, existe $\lambda\in\mathbb{R}$ tal que $u=\lambda v$, luego $a_k^{1/2}=\lambda a_k^{3/2}$ para $1\leq k\leq n$. Elevando al cuadrado, $a_k^3=\lambda^2a_k$ y, sumando en $k$, ha de cumplirse que $216=96\lambda^2$, de donde $\lambda=\frac{3}{2}$. Por tanto, $a_k^3=\frac{9}{4}a_k$ de donde tiene que ser $a_k=\frac{3}{2}$ para todo $k$ y, finalmente, tenemos que $n=64$. Es fácil ver que esta sucesión constante verifica las tres ecuaciones del enunciado.
Si crees que el enunciado contiene un error o imprecisión o bien crees que la información sobre la procedencia del problema es incorrecta, puedes notificarlo usando los siguientes botones:
Informar de error en enunciado Informar de procedencia del problema
Problema 149
Dado $n\geq 1$, hallar todas las $n$-uplas de números reales $x_1,x_2,\ldots,x_n\geq 1$ tales que \[\sqrt{x_1}+\sqrt[3]{x_2}+\ldots+\sqrt[n+1]{x_n}=\sqrt{n}\sqrt{x_1+x_2+\ldots+x_n}.\]
pistasolución 1info
Pista. Realmente, para cualesquiera $x_1,x_2,\ldots,x_n\geq 1$, una de las desigualdades en la igualdad del enunciado se cumple, luego el problema puede reducirse a estudiar cuándo se alcanza la igualdad en una desigualdad. La desigualdad de Cauchy-Schwarz puede ser útil para terminar el problema.
Solución. Evidentemente, si tomamos $x_1=x_2=\ldots=x_n=1$, tenemos una solución. Veremos ahora que, independientemente de los valores de las variables siempre se tiene una desigualdad $\leq$ en la expresión del enunciado y veremos que la igualdad sólo se alcanza para esta solución.

Aplicando la desigualdad de Cauchy-Schwarz a los vectores \begin{eqnarray*} u&=&\left(\sqrt{x_1},\sqrt[3]{x_2},\ldots,\sqrt[n+1]{x_n}\right)\\ v&=&\left(1,1,\ldots,1\right) \end{eqnarray*} llegamos a que \[\sqrt{x_1}+\sqrt[3]{x_2}+\ldots+\sqrt[n+1]{x_n}\leq\sqrt{n}\sqrt{x_1+x_2^{2/3}+x_3^{2/4}+\ldots+x_n^{2/(n+1)}}\] Ahora bien, como $x_2,\ldots,x_n$ son mayores o iguales que uno que uno, se tiene que $x_k^{2/(k+1)}\leq x_k$ pues $\frac{2}{k+1}\leq 1$. Esto demuestra la desigualdad que queremos y, si la igualdad se alcanza, entonces $x_2=\ldots=x_n=1$ por la última desigualdad y $x_1=1$ por la igualdad en la desigualdad de Cauchy-Schwarz. Deducimos que $x_1=x_2=\ldots=x_n=1$ es la única solución al problema.

Si crees que el enunciado contiene un error o imprecisión o bien crees que la información sobre la procedencia del problema es incorrecta, puedes notificarlo usando los siguientes botones:
Informar de error en enunciado Informar de procedencia del problema
Problema 148
Dados tres números reales positivos $x,y,z\geq 0$ tales que $x+y+z=1$, demostrar que \[\left(1+\frac{1}{x}\right)\left(1+\frac{1}{y}\right)\left(1+\frac{1}{z}\right)\geq 64.\] Analizar para qué valores de $x,y,z$ se obtiene la igualdad.
pistasolución 1info
Pista. Manipula usando la igualdad $x+y+z=1$ y utiliza la desigualdad entre las medias aritmética y geométrica.
Solución. Cambiando el $1$ de los numeradores de las fracciones por $x+y+z$ y operando, la desigualdad a probar es equivalente a \[(2x+y+z)(x+2y+z)(x+y+2z)\geq 64xyz.\] Ahora bien, usando repetidamente la desigualdad entre las medias aritmética y geométrica, tenemos que \begin{eqnarray*} 64xyz=64\sqrt{xy}\sqrt{yz}\sqrt{xz}&\leq& 8(x+y)(y+z)(x+z)\\ &=&8\sqrt{(x+y)(y+z)}\sqrt{(x+y)(x+z)}\sqrt{(x+z)(y+z)}\\ &\leq&(x+y+y+z)(x+y+x+z)(x+z+y+z)\\ &=&(2x+y+z)(x+2y+z)(x+y+2z) \end{eqnarray*} con lo que la desigualdad está probada. La igualdad se alcanza si, y sólo si, $x=y=z=\frac{1}{3}$.
Si crees que el enunciado contiene un error o imprecisión o bien crees que la información sobre la procedencia del problema es incorrecta, puedes notificarlo usando los siguientes botones:
Informar de error en enunciado Informar de procedencia del problema
Problema 105
Supongamos que $a,b,c,d\in\mathbb{R}$ son números positivos que satisfacen la condición $ab+bc+cd+da=1$. Demostrar que \[\frac{a^3}{b+c+d}+\frac{b^3}{a+c+d}+\frac{c^3}{a+b+d}+\frac{d^3}{a+b+c}\geq\frac{1}{3}\] y analizar en qué casos se alcanza la igualdad.
pistasolución 1info
Pista. Utilizar la desigualdad de Chebyshev para transformar la desigualdad del enunciado.
Solución. Supondremos sin perder generalidad que $a\geq b\geq c\geq d\gt 0$ y llamaremos $E$ al miembro de la izquierda de la desigualdad del enunciado. Si definimos \[x_1=a^3,\quad x_2=b^3,\quad x_3=c^3,\quad x_4=d^3,\] \[y_1=\frac{1}{b+c+d},\quad y_2=\frac{1}{a+c+d},\quad y_3=\frac{1}{a+b+d},\quad y_4=\frac{1}{a+b+c},\] se cumple que $x_1\geq x_2\geq x_3\geq x_4\gt 0$ e $y_1\geq y_2\geq y_3\geq y_4\gt 0$. Por tanto, podemos aplicar la desigualdad de Chebyshev a estos números y obtenemos que \[E\geq\frac{1}{4}(a^3+b^3+c^3+d^3)\left(\frac{1}{b+c+d}+\frac{1}{a+c+d}+\frac{1}{a+b+d}+\frac{1}{a+b+c}\right).\] La desigualdad entre las medias cúbica y aritmética aplicada a los $x_i$ nos dice que \[\frac{1}{4}(a^3+b^3+c^3+d^3)\geq\frac{1}{64}(a+b+c+d)^3\] y la desigualdad entre las medias aritmética y armónica aplicada a los $y_i$ que \[\frac{1}{b+c+d}+\frac{1}{a+c+d}+\frac{1}{a+b+d}+\frac{1}{a+b+c}\geq\frac{16}{3(a+b+c+d)}.\] Usando estas dos últimas desigualdades, llegamos a que \[E\geq\frac{16(a+b+c+d)^3}{3\cdot 64(a+b+c+d)}=\frac{1}{12}(a+b+c+d)^2.\] Finalmente, usando que la condición del enunciado se escribe como $(a+c)(b+d)=1$ y usando la desigualdad entre las medias aritmética y geométrica, obtenemos \[(a+b+c+d)^2\geq 4(a+c)(b+d)=4,\] con lo que $E\geq\frac{1}{3}$ como queríamos probar. Si la igualdad se alcanza, entonces en la desigualdad entre las medias cúbica y aritmética para los $x_i$ se deduce que $a=b=c=d$ y , por la condición $ab+bc+cd+da=1$, estos cuatro números tienen que ser iguales a $\frac{1}{2}$, Se comprueba que, para esa elección, se alcanza la igualdad luego esa es la única solución.
Si crees que el enunciado contiene un error o imprecisión o bien crees que la información sobre la procedencia del problema es incorrecta, puedes notificarlo usando los siguientes botones:
Informar de error en enunciado Informar de procedencia del problema
José Miguel Manzano © 2010-2024. Esta página ha sido creada mediante software libre